Sei sulla pagina 1di 17

388 Statistical Mechanics

change in temperature scale; it is sho:vn as the dotted curve of Figure


27-3b. And finally the metastable mixture of i orthodeuterium plus 1
paradeuterium, obtained by cooling deuterium gas without the presenc!
of a catalyst, is given by the formula
[
2 a2 1 a
2
]
Cvt = NkT 3 aT
2
(T ln z;ot) + 3 aT
2
(T ln z ~ a t (27-23)
(metastable mixture)
and is shown, for Orat = 43K, as the solid line of Figure 27-3b. All of
these curves are verified by experimental data, taken under the different
conditions, appropriate for the three cases.
Thus the effects of quantum statistics turn up in odd corners of the
field, at low temperatures and for substances a part of which can
gaslike to low enough temperatures for the effects of degeneracy to become
evident. For the great majority of substances and over the majority of
the range of temperature and density, classical statistical mechanics is
valid, and the calculations using the canonical ensemble of Chapters 19
through 22 quite accurately portray the observed results. The situations
where quantum statistics must be used to achieve concordance \Vith
experiment are in the minority (luckily; otherwise our computational
difficulties would be much greater). But, when they are all considered,
these exceptional situations add up to exhibit an impressive demonstra-
tion of the fundamental correctness of quantum statistics. They are
some of the aspects of reality, mentioned at the beginning of Chapter 16,
the explanation of which is the sole justification for the theoretical super-
structure of statistical mechanics discussed in these last twelve chapters.
PROBLEMS
Chapter 2
2-l. The amount of heat !::.Q added to a unit mass of some homogeneous
material raises its temperature by an amount !::.T, where !::.Q = cmi::.T, Cm being
the heat capacity of the material per unit mass. A temperature gradient in the
material will produce a heat flow J heat units per second per unit area perpendicu-
lar to the flow. The relationship is J = - Kr grad T, where Kr is the thermal
conductivity of the material. Use the equation of continuity for fluid flow to
relate the divergence of J to the time rate of change of Q and therefore of T.
From this obtain the differential equation for the temperature distribution in
the material, as a function of x, y, z, and t, in terms of em, K1, and p, the density of
the material.
2-2. Suppose that all the atoms in a gas are moving with the same
speed v, but that their directions of motion are at random.
(a) Average over directions of incidence to compute the mean number
of atoms striking an element of wall area dA per second (in terms of N, V, v, and
dA) and the mean momentum per second imparted to dA.
(b) Suppose, instead, that the number of atoms having speeds between
v and v + dv is 2N[1 - (vfvm)](dvfvm) for v < Vm (the directions still at random).
Calculate for this case the mean number per second striking dA and the mean
momentum imparted per second, in terms of N, V, vm, and dA. Show that
Eq. (2-4) holds for both of these cases.
2-3(a) For a perfect gas of point particles the mean kinetic energy per
particle (K.E.)tran is a function f(T) of the temperature T. Thus any change in
temperature will produce a change in the internal energy U of the gas. But an
addition of heat !::.Q produces an increase !::.T in temperature; by definition the
relationship is !::.Q = C !::.T, where Cis the heat capacity of the gas. Therefore,
for the gas of Eq. (2-1), an addition of heat is equivalent to an increase in internal
energy of the gas. If !::.Q is expressed in the same energy units (joules) as U,
what is the equation relating em, the heat capacity per unit mass of the gas, to
N m, the number of molecules per unit mass and f', the slope of f( T)?
(b) When there is a temperature gradient in a substance there cannot
be thermodynamic equilibrium because heat is flowing (see Problem 2-1). There
can be mechanical equilibrium, however, if the pressure is uniform throughout
the material. Suppose the material is a perfect gas of point particles, and sup-
pose the temperature gradient is small enough so that Eq. (2-4) holds at each
point in the gas, with (K.E.)tran = f(T). What must be the value of the gradient
of the density p of the gas, in terms of the gradient of T, of p, em and U, for there
to be mechanical equilibrium?
389
390
Thermodynamics
Chapter 3
3-l. The coefficient of thermal expansion {3 and the compressibility K
of a substance are defined in terms of partial derivatives
f3 = (1/V)(av;aT)p K = -(1/V)(av;aP)r
(a) Show that
(a{3jaP)r = -(aKjaT)p
and that
({3/K) = (aP;aT)v
for any substance.
(b) What are the values of {3 and K and (aPjaT)v for a perfect gas?
For a substance with the virial equation of state given by Eq. (3-5)?
3-2. It is found experimentally that, for a given gas,
RVZ(V- nb)
{3 = R=T=v::-;:a-_---'="2-an-(:-:oV=--'---n--=b:-:-:)2
VZ(V- nb)2
K =
nRTV3 - 2an
2
(V - nb)
2
where a and b are constants, and also that the gas behaves like a perfect gas for
large values of T and V. Find the equation of state of the gas.
3-3. Partial derivatives analogous to the coefficient of thermal expan-
sion and the compressibility can be defined for the magnetic equation of state,
am=
Find the expressions for these quantities for substances obeying Curie's law,
Eq. (3-8). Also find (aS)jaT) 911 .
3-4. A gas obeys equation of state (3-4). Show that for just one
critical state, specified by the values T, and V" both (aPjaV)r and (a
2
PjaV
2
)r
are zero. Write the equation of state giving P jP, in terms of T T, and V V,.
Plot three curves for P jP, as a function of V V,, one forT = one for T = T"
and one for T = 2T,. What happens physically when the equation indicates
three allowed values of V for a single P and T?
3-5. From Figure 3-3 estimate the volume of a mole of COz at 300K
and 20 atmospheres. A mole of steam at 600K occupies 1.4 cubic meters; what
is its pressure in atmospheres?
3-6(a) By considering the charging of a parallel-plate condenser,
calculate the work done by a homogeneous, isotropic dielectric in terms of the
volume v of the dielectric and the uniform electric field applied, when the
electric polarization of the dielectric changes by dll3.
(b) It is found experimentally that
= [T/(AT +B)]
and = [Bil3/(AT + B)2]
Compute ('all3jaT) 1\: and, by integration, obtain the equation of state, ll3 as a
function of and T.
3-7. Express the van der Waals equation of state (3-4) in the form of
the virial equation (3-5). Find B(T) and C(T) in terms of a, b, and T.
Problems 391
Chapter 4
4-I. A gas with van der Waals equation of state (3-4) has an internal
energy
U = jnRT- (an
2
/V) + U0
Compute Cv and CP as functions of V and T and computeT as a function of V
for an adiabatic expansion.
4-2. An ideal gas for which Cv = tnR and (aU jaV)r = 0 is taken
from point a to point bin Figure P-1, along three paths, acb, adb, and ab, where
P2 = 2PJ, V2 = 2VJ.
(a) Compute the heat supplied to the gas, in terms of n, R, and T1, in
each of the three processes.
(b) What is the heat capacity of the gas, in terms of R, for the process
ab?
FIGURE P-1
. 4-3. A solid, obeying Eqs. (3-6), (3-8), and (4-8) and
havmg a heat capacity C.911 = nAT
3
, is magnetized isothermally (at constant
volume) at temperature To from 9)( = 0 to a maximum magnetic field of
How much heat must be lost? It is then demagnetized adiabatically (at con-
volume) to IJJC = 0 again. Compute the final temperature T1 of the solid,
m terms of S)m, To, A, and D. How do you explain away the fact that, if S)m is
large enough or To small enough, the formula you have obtained predicts that
T1 should be negative?
4-4. Derive equations for (aU ;aT)P and (aU ;aP)r analogous to
(4-4) and (4-6). Obtain an expression for (aHjaP)r. For a perfect gas,
wrth. CP = tnR and (CP- Cv)j(aVjaT)p = P, integrate the partials of H to
obtam the enthalpy.
4-5. Figure P-2 shows a thermally isolated cylinder, divided into two
parts by a thermally insulating, frictionless piston. Each side contains n moles
of a perfect gas of point atoms. Initially both sides have the same temperature;
392
Po
To
Vo
FIGURE P-2
Po
To
Vo
Thermodynamics
heat is then supplied slowly and reversibly to the left side until its pressure has
increased to (243P o/32).
(a) How much work was done on the gas on the right?
(b) What is the final temperature on the right?
(c) What is the final temperature on the left?
(d) How much heat was supplied to the gas on the left?
4-6. The ratio CP/Cv for air is equal to 1.4. Air is compressed from
room temperature and pressure, in a diesel engine compression, to 1/15 of its
original volume. Assuming the compression is adiabatic, what is the final
temperature of the gas? . . .
4-7. Find the relationship between T and V for the reversible, adiabatic
compression of a gas obeying van der Waals equations (3-4) and having a heat
capacity at constant volume C. which is independent of T and of V. Use the
value of U given in Problem 4-1 to obtain Cp and thence 'Y = (Cp/C.).
4-8. A gas, for which (oU jaV)r = 0, satisfies the virial equation of
state (3-5), with B and C independent of T. . . .
(a) How much heat must be supplied to expand the gas, m a quas1statlc,
isothermal process, from volume V1 to volume Vz? ..
(b) Express the difference (CP - Cv) between the heat capamtles of
this gas in terms of T and V.
4-9. At low temperatures the heat capacity at constant volume of
NaCl is proportional to the third power ofT in degrees Kelvin; C. = 0.0874 n ra
joulesjkg-deg K.
(a) At high temperatures C. = 3nR approximately. Compare the
high-temperature value with the value at T = 50K.
(b) How much heat is required to raise the temperature of a mole of
NaCl from 25K to 50K?
Chapter 5
5-I. Compute

and

for a Carnot's cycle using a perfect gas


of point particles in terms of nR and Th and T.. Using the perfect-gas scale of
temperature that W
23
= - W
41
Show that the efficiency of the cycle
is (Th - T.)./Th and thus prove that the perfect-gas temperature scale coincides
with the thermodynamic scale of Eq. (5-5).
5-2. A magnetic material satisfying Eqs. ( 4-8) and (3-8) has a.
heat capacity, Cv!ln = C. It is carried around a Carnot cycle shown m !1gur:e
P-3 ffiC being reduced isothermally from ffi11 to ffiCz at Th, then reduced adiabatl-
from 9)C
2
to fiCa, when it has temperature T., then remagnetized isothermally
at T. to ffi1
4
, and thence adiabatically back to Th and ffi11.
Problems
3
0

FIGURE P-3
393
(a) Express fiCa in terms of Th, T., D, C, and ffi12 and relate ffi1 4
similarly with ffiC,.
(b) How much heat is absorbed in process 1-2? How much given off
in process 3-4?
(c) How much magnetic energy aW is given up by the material in
each of the four processes? Show that aW 2a = - aW 41
(d) Show that the efficiency of the cycle heat-magnetic-energy is
(Th- T.)/Th.
5-3. When a mole of liquid is evaporated at constant temperature T
and vapor pressure P .(T), the heat absorbed in the process is called the latent
heat Lv of vaporization. A Carnot cycle is run as shown in Figure P-4, going
isothermally from 1 to 2, evaporating n moles of liquid and changing volume from
VI to v2, then cooling adiabatically to T - dT, Pv - dPv by evaporating an
additional small amount of liquid, then recondensing the n moles at T - dT,
from Va to V4, and thence adiabatically to Pv, T again.
(a) Show that V2 - V1 = V
0
- Vz where V
0
is the volume occupied
by n moles of the vapor, and Vz the volume of n moles of the liquid.
(b) Find the efficiency of the cycle, in terms of dPv, V
0
- Vz, and nLv.
(c) If this cycle is to have the same efficiency as any Carnot cycle, this
P. T V
t
p
vapor
_...,..._ ... ..___.. ____ ---
- - --- -
liquid::_::::
-- -- =---=- --
T 2
Pv
\ \
Pv- dPv
4 3
T dT
VI v2

FIGURE P-4
.394
Thermodynamics
efficiency must be equal to (Th - Tc)/Th = dT jT. Equating the two expres-
sions for efficiency, obtain an equation for the rate of change dPv/dT of the
vapor pressure with temperature in terms of Va- V1, n,.Lv, ar:d T.
5-4. In the tropics the water near the surface IS considerably warmer
than the deep water. Would an engine operating between these two levels
violate the second law?
5-5. Electromagnetic radiation in an evacuated vessel of volume V,
at equilibrium with the walls at temperature T, behaves like a gas of
having internal energy U and pressure P: U = aVT
4
, P = .fsaT4, where a IS
Stefan's constant [see Eq. (7-8)].
(a) Find the relation between P and V for an adiabatic process and
the relationship between heat absorbed and change of volume in an isothermal
process, for this gas.
(b) Plot the closed curve, on the P - V plane, for a Carnot cycle.
(c) Compute the heat absorbed and the work done in each part of the
cycle and prove that the efficiency is [(Th- Tc)/Th].
5-6. Prove that it is impossible for two quasistatic adiabatics to inter-
sect. (Hint: show that the second law is violated if they do.)
5-7. Suppose we desire to use a temperature scale r such that the
efficiency deficit function' of Eq. (5-2) is a subtractive function of r, 'l'(rh - rc).
(a) Show that the thermodynamic temperature T is an exponential
function of r.
(b) If we wish r to be zero at the melting-point of ice and to be 100 at
the boiling point of water, what is the exact functional relationship between T and
r? What value of r corresponds to T = 0?
(c) What are the equation of state and the heat capacities Cv and CP
for a perfect gas of point atoms in terms of the r scale?
5-8. An ideal gas, satisfying Eqs. ( 4-7) and ( 4-12) is carried around the
cycle shown in Figure P-5; 1-2 is at constant volume, 2-3 is adiabatic, 3-1 is at
constant pressure, V
3
is 8 V
1
, and n moles of the gas are used.
(a) What are the heat input, heat output, and efficiency of the cycle,
in terms of PI, vb n, and R?
(b) Compare this efficiency with the efficiency of a Carnot cycle
operating between the same extremes of temperature.
Chapter 6
6-l. An amount of perfect gas of one kind is in the volume C1 V of
Figure P-6 at temperature T and pressure P, separated by an impervious dia-
Problems
P, T,
c1v
D
P, T,
c2v
FIGURE P-6
395
phragm D from a perfect gas of another kind, in volume C2V and at the same
pressure and temperature (C1 + C2 = 1). The volume Vis isolated thermally.
What is the entropy of the combination? Diaphragm D is then ruptured and
the two gases mix spontaneously, ending at temperature T, partial pressure C1P
of the first gas, C
2
P of the second gas, all in volume V. What is the entropy now?
Devise a pair of processes, using semipermeable membranes (one of which will
pass gas 1 but not 2, the other which will pass 2 but not 1), which will take the
system from the initial to the final state reversibly and thus verify the change in
entropy. What is the situation if gas 1 is the same as gas 2?
6-2. Two identical solids, each of heat capacity Cv (independent of
T), one at temperature T + t, the other at temperature T- t, may be brought
to common temperature T by two different processes:
(a) The two bodies are placed in thermal contact, insulated thermally
from the rest of the universe and allowed to reach T spontaneously. What is
the change of entropy of the bodies and of the universe caused by this process?
(b) First a reversible heat engine, with infinitesimal cycles, is operated
between the two bodies, extracting work and eventually bringing the two to
common temperature. Show that this common temperature is not T, but
V(T
2
- t
2
). What is the work produced and what is the entropy change in
this part of process b? Then heat is added reversibly to bring the temperature
of the two bodies to T. What is the entropy change of the bodies during the
whole of reversible process b? What is the change in entropy of the universe?
6-3. Show that T dS = Cv dT + T(aPjaT)v dV and that T dS =
Cv(aTjaP)v dP + CP(aTjaV)P dV.
6-4. A paramagnetic solid, obeying Eqs. (3-6), (3-8), and ( 4-8), has a
heat capacity Cpo(T) (at zero magnetic field) dependent solely on temperature.
First, show that
T dS = Cps;, dT- T(aVjaT)p dP + T(aiJJtjaT)s;,
and, analogous to Eq. (8-13), that = T(a
2
1JJtjaT
2
)s;,. From this,
show that
S = loT (Cpo/T) dT - - {3VoP
and thence obtain energy U as a function of T, P, ObtainS as a function
of T, V, and IJJt and thence obtain U as a function of T, V, and IJJt.
6-5. Express the entropy and the volume of a perfect gas of point
atoms in terms of its pressure and temperature. Then integrate the Gibbs-
Duhem Eq. (6-9) to obtain the chemical potentialp. of a mole of the gas.
6-6. At very low temperatures the entropy, magnetization, and internal
energy of the atomic magnets in a paramagnetic crystal are fairly accurately
396 Thermodynamics
represented by the equations
where
and where Rm, {3, and T m = 0.2K are constants.
(a) Compute (aff)jaT)IJJI and thence find (aUmja'illC)r as a function of
T. Plot it as a function ofT for 0 < T < 5T m for fO = 0 and for f3f0 = 2.
What does this predict regarding the relation between heat absorbed and rate of
gain of u m in an isothermal process?
(b) Plot Sm as a function of T for 0 < T < 5T m for fO = 0 and
= 2. Use these curves to recapitulate the arguments regarding the third
law of thermodynamics. Roughly, what temperature can be reached by starting
at T = 1 K, fO = 0, increasing to f3f0 = 2 isothermally and then reducing to
fO = 0 isentropically? How much further could you go in two more such steps?
How much in eight more steps?
Chapter 7
7-l. A gas obeys the virial equation of state (3-5) with B and C
independent of T and its heat capacity at constant volume C. = lnR. Compute
the Joule-Thomson coefficient (aTjaP)H as a function ofT and V. Does this
gas have an inversion point?
7-2. The Stirling cycle consists of two isothermal processes, at Th and
Tc and two processes at constant volume, as is shown in Figure P-7. The
characteristics of the cycle depend on how the heat is transferred to and from
the engine during the constant-volume parts of the cycle.
(a) Consider first the case where only two heat reservoirs are aYailable,
at temperatures Th and Tc. Find the entropy change of the universe for
41 of the cycle.
(b) Show that this change is always positive.
t
p
0
2
3
v-
FIGURE P-7
Problems
397
(c) the use the word for the fraction (net work)/ llQ
12
of Eq. (5-1) fo: this cycle. Fmd the efficiency, so defined, for this cycle, assuming
that the workmg substance is a perfect gas of point atoms.
(d) Now suppose that a continuous set of heat reservoirs, for every
temperature between Tc and Th, is available, so that 23 and 41 can be traversed
reversibly. What now is the efficiency of the cycle?
(e) In view of your answers to (c) and (d), explain the detrimental
effect of the increase in entropy in the former case.
7-3. A .ve:y set of formulas for the entropy and density
of the normal flmd helmm II IS sn = 1600 joules/kg deg (which thus equals
the entropy per umt mass of the whole fluid sx at the transition point) and
Pn = p(T/Tx)
4
, where P = Pn + Ps = 144 kg/m
3
is the density of the whole
fluid and where Tx = 2.2K is the temperature of the transition point.
(a) Enough heat is supplied to the coil H of Figure 7-2 to make the
temperature 1.2K, whereas the temperature outside the vessel is 1.1 K. What
is the pressure difference between the inside and outside of the vessel, in
atmospheres?
(b) Using the formula

plot the velocity of second


sound versus T for the range iTx < T < Tx.
. 7-4. Show: that, when temperature T and applied magnetic field ff) are
the mdependent vanables, the heat capacity for constant fO is Cs;;, = (aHmjaT),
where Hm = Um- and thus that Hm is the magnetic heat content of the
material at constant applied field.
. (a) Find Cs;;, for a material obeying Curie's law and having a heat
capacrty for constant magnetization CIJJI = [nRmT
2
/(02 + T2)]. What is the
entropy of this material? Can this formula be correct down to T = 0?
(b) Find Cs;;, for the material of Problem 6-6. Plot it, as a function of
T for 0 < T < 5T m, for f3ff) = 0 and for f3f0 = 2. Show that for T much greater
than both T m and {3f{)T m this material obeys Curie's law and that its Cs;;, is
inversely proportional to T2.
Chapter 8
. 8-l. The tension J in a plastic rod, stretched to a length L, at tempera-
ture T, IS J =: A(T)(L - Lo), where Lois independent of T. Its heat capacity
at constant IS CL = C(T)- M(T)N(L), where N = 0 when= L
0
and where
MN Cover the range of interest of T and L.
. . (a) . Show M(T)N(L) = iTA"(T)(L - L
0
)
2
, where the primes on
A mdicate with respect to T. What is the heat capacity of the
rod at constant tension?
(b) Compute the entropy of the rod, as function of T and L, if S 0
as T 0. What does this require of function A (T)?
. . (c) the rate of change (aT;aL), of the temperature of the rod for
adiabatic stretchmg. If the temperature rises when the rod is stretched adiabati-
cally, does the.en.tropy increase or decrease as the rod is stretched isothermally?
What does this mdicate about the change in the amount of disorder of the
molecules of the rod on stretching?
(d) Compute the internal energy U of the rod and then compute
F = U - TS as a function of T and L. Calculate the partials ofF with respect
to T and Land show that they equal -Sand J, respectively.
398 Thermodynamics
8-2. A gas obeys the van der Waals equation of state (3-4) and has
heat capacity at constant volume Cv = jnR. Write the equation of state in
terms of the quantities t = T /T., p = P /P., and v = V jV., where T. = 8ajZ7 Rb,
P. = ajZ7b
2
, V. = 3nb (see Problem 3-Z). Calculate T.S/PY. in terms of t
and v, likewise F /P. V. and G/P. V.. Fort = t plot pas a function of v from the
equation of state. Then, for the same value of t, calculate and plot G/P. V. as a
function of p, by graphically finding v for each value of p from the plot, and then
computing G/P. V. for this value of v (remember that for some values of p there
are three allowed values of v). The curve for GjP. V. crosses itself. What is the
physical significance of this?
8-3. The work done by a homogeneous, isotropic dielectric for a small
change of electric polarization is - dlj3, where is the applied electric field
strength and lj3 is the polarization, if the field is uniform.
(a) Find the Maxwell relations for this case. Assume constant V.
(b) Find the heat supplied to the dielectric when a parallel-plate
condenser is charged isothermally to a final field strength The dielectric
obeys the law lj3 = [a + (b/T) Interpret your result in terms of entropy and
disorder.
8-4. The Gibbs function of a nonideal gas is
G = nRT ln P + P[nb- (najRT)] + j(T)
where a and bare constants and f is a function of T only. Find the equation of
state and show that it agrees with the van der Waals equation of state if second-
order terms in a and b are neglected. Compute the heat capacity at constant
pressure.
8-5. Find the magnetic Gibbs function and the magnetic Helmholtz
function for the material of Problem 6-6. Calculate expressions for the partials
(aTjaf)). and (aTjaf))u as functions ofT and f).
8-6. The equation of state and the heat capacity of a gas are
P = RT[(njV) + (njV)
2
B(T)]
d
Cv = jnR - (n
2
R/V) dT (T
2
B')
where B'(T) = (dBjdT) and where the second term in the brackets is small
compared to the first term for the useful range of T and V.
(a) Show that the second term in the expression for Cv must be
present if the equations are to be self-consistent. Compute Cw
(b) ComputeS and U and show that (aU/aV)r is not zero.
(c) Compute F, H, and Jl., all as functions of T and V.
8-7. The Helmholtz function for a homogeneous, isotropic solid is
F = (1/ZKoVo)(V- Vo)2 + 3nRT ln { 1- exp (- [1 + a(Vo- V)]}
where R, Ko, Vo, a, and Oo are independent of T and V.
(a) Compute the equation of state and thence find the compressibility
and the coefficient of thermal expansion
K= -(1/V)(av;aP)r f3 = (1/V)(av;aT)p
Problems
399
(b) Compute the entropy S and the internal energy U of the solid.
Plot (Sj3nR) as a function of (T/Oo) from 0 to Z, for a(Vo- V) = 0 and = 0.5.
Is this set of formulas consistent with the third law?
(c) Calculate the expressions for (aTjaV)u and (aTjaV)s.
(d) Compute the heat capacities Cv and Cp. What are their limiting
formulas for T 00 and T >> ()
0
?
8-8. Express the entropy of the gas of Eq. (8-ZZ) in terms of n, U,
and V, rather than n, T, and V. Then show that the entropy parameters
Fo = (aSjaU)vn = (1/T), Fv = (aSjaV)un = (P/T), and Fn = (aSjan)uv
= (Jl./T).
8-9. Use Eqs. (8-48) to show that, when electric current J. flows
through a junction between a w!re of material A to one of material B, all kept
at constant temperature T, heat Js evolved.
8-lO(a) Show that the enthalpy H = G- T(aGjaT)P where G is the
Gibbs function. Thus show that q in Eq. (8-38) would be zero' if the thermo-
osmotic process were reversible.
. The gas diffusing through the porous partition is a perfect gas of
partwl.es. From Eqs. (8-4Z) devise an experiment to measure q indirectly
without havmg to measure Q or K or grad P. What is the formula for q in terms
of the quantities measured?
Chapter 9
9-l. Assume that near the triple point the latent heats L and L
are independent of P, that the vapor has the equation of state of a pe;fect gas:
that the volume of a mole of solid or liquid is negligible compared to its vapor
and that the difference Vz - V. is positive, independent of P or T, and
IS small compared to nLm/T. Using these assumptions interrrate the three
Clausius-Clapeyron equations for the vapor-pressure sublimati;n-pressure and
melting-point curves. Sketch the form of these on the P-T plane. '
. 9-2. The heat of fusion of ice at its normal melting point is 3.3 X 105
JOules/kg specific volume of ice is greater than the specific volume of
water at this pomt by 9 X I0-
5
m
3
/kg. The value of (1/V)(aVjaT)P for ice is
16 X 10-
5
per degree and the value of -(1/V) X (aVjaP)T is lZ x 10-11
m
2
/newton.
(a) at -zoe and at atmospheric pressure is compressed iso-
thermally. Fmd the pressure at which the ice starts to melt.
(b) Ice at -2'C and atmospheric pressure is kept in a container at
constant volume and the temperature is gradually increased. Find the tem-
perature at which the ice begins to melt.
(c) Ice at -zoe and atmospheric pressure is compressed adiabatically.
At what pressure will the ice melt?
9-3. Obtain an expression for the fraction (flU /nL) of the latent heat
of evaporation, which goes into a chancre of internal eneray in the chanoe of
phase, in terms of Pv, T, and (dPv/dT). What is the value of this fraction the
transition from water to ice at 0C?
9-4. The of ice, near the melting point, is Z100 joules/kg
deg and that of water IS 4ZOO JOUles/kg deg. How much does the heat of fusion
of ice change with an increase of pressure of 10 atm?
?-5. !t mass m of water, originally at temperature Th, is placed inside
an electnc refngerator. The refrigerator operates between the water and the
400 Thermodynamics
room at T which may be considered as a heat reservoir. The water is frozen
and the icehis cooled further to a temperature To. The sp.ecific heats Cw and c;
of water and ice may be considered as a con;;tant over this temperat_ure range.
Find the minimum amount of elec.trical that must b: to the
refrigerator for this process. What IS the numencal value of this mJmmal energy,
in kilowatt-hours, if m = 1 kg, Th = 27aC, and To= -23C? . .
9-6. The temperature of a long vertical column of a certam matenal
is everywhere equal. When this is To the. a certain
point in the column is found to be that pomt IS When
the temperature is reduced by 0.2 C the sohd-hqmd IS to
shift upward by 0.5 m. The heat of fusion of the
and the density of the liquid is 1000 kg/m
3
What IS the of the solid. m
the vicinity of the interface? (Hint: the pressure at the ongmal mterface pomt
is not changed by the temperature change.)
9-7. Show that = (Tc/nLc)(Vn- V.) for a superconductor,
where T is the transition temperature at pressure P, and where Vn and V, are the
volumesc of the normal and the superconducting phases. .
9-8. The magnetic field .Pc for superconductive transition, for a
particular material, depends on T as follows:
.Pc = 3) 0[1- (1- a)(T/To)
2
- a(T/To)3]
Its heat capacity, in the superconductive phase, at very low temperatures, goes
to zero quadratically with T,
Cc = {32V(T/To)2 + f3aV(T/To)3 +
Show that the heat capacity of the normal phase (for 5) > .Pc) at low tempera-
tures must depend on T as follows:
Determine the values of 'Y1 and 'Y2 in terms of J.l.o, To, .Po, a, and f12
Chapter 10
IO-l. The observed equilibrium constant K(T) for a gaseous reaction
is usually expressed in the series form
ln K(T) = (QjT) +A In T + B + CT +
Show that Q is the heat evolved in the reaction at zero temperature.
10-2. The coefficients in the expansion for ln K (see Problem 10-1)
for the iodine dissociation reaction 2I I2 are Q = 1!400, :4- = 0.75, B = 1,
C, D, etc., are negligible, where Tis in K and the of K are (atm) .
What fraction of the iodine is dissociated at 1 atm and 500 K? At what rate IS
this fraction changing with T? .
10-3. The equilibrium constant for the reactiOn 2 SO 2 SOz + 02
has the value 0.29 atm at T = 1000K. If one mole of S02 and 2 moles of 02
are introduced into a vessel and maintained at P = 4 atm and T = 1000, find
the number of moles of SOa present at equilibrium.
Problems
401
Chapter II
II-I. Calculate the mean value (n) and variance of n for the
following discrete probability distributions:
Pn = (1 - a)an
Geometric
Binomial
Poisson
Pn = [N!jn!(N- n)!]pn(l- p)N-n
Pn = (Nnjn!)e-N
11-2. Calculate the mean value (x) and variance u; of x for the
continuous distributions, having the following probability densities:
Exponential f(x) = (1/f-)e-zii-
Erlang f(x) = (4xjf-2)e-2z!A
ll-3. The probability that a certain trial (throw of a die or drop of a
bomb, for example) is a success is p for every trial. Show that the probability
that m successes are achieved in n trials is
n!
Pm(n) = pm(l- p)n-m
m!(n- m)!
(this is the binomial distribution)
Find the average number iii of successes inn trials, the mean-square (ili2) and the
standard deviation !:J.m of successes in n trials.
ll-4. The probability of finding n photons of a given frequency in an
enclosure that is in thermal equilibrium with its walls is Pn = (1 - a)an, where
a(O < a < 1) is a function of temperature, volume of the enclosure, and the
frequency of the photons. What is the mean number n of photons of this fre-
quency? What is the fractional deviation !:J.n/n of this number from the mean?
Express this fractional deviation in terms of n, the mean number. For what
limiting value of n does the fractional deviation tend to zero?
ll-5(a) Show that the number of different ways in which M dis-
tinguishable objects can be placed in C numbered boxes, with no restriction on the
number of objects per box, is CM.
(b) Show that the number of different ways in which M distinguishable
objects can be placed inC numbered boxes, with no more than one object per box
is [C!j(C - M) !], where M ::::; C. '
(c) Show that the number of ways in which iff indistinguishable
objects can be placed inC numbered boxes, with no more than one object per box
is (C!/M!(C - M) !], where M ::::; C.
(d) Show that the number of ways in which M indistinguishable objects
can be placed in C numbered boxes, with no restriction on the number of objects
per box, is ((C + M - 1) !/M!(C- 1) !].
(e) Show that the number of ways in which M indistinguishable
objects can be placed in C numbered boxes, with at least one object per box, is
[(M - 1) !j(C- 1) !(M - C)!], where (M C).
ll-6. Aircraft arrive at an airport at times that are more or less
randomly distributed in time. Suppose the probability density that the next
arrival occurs at a timet after the previous arrival is a(t).
(a) Show that the probability that no arrivals occur in a time t after
the last one is
Au(t) = J"' a(x) dx
402
Kinetic Theory
where A
0
(0) = 1, and that
T = Ia"' ta(t) dt = Ia"' A 0(t) dt
is the mean time between arrivals. Compute Ao(t) for an exponential distribu-
tion, a(t) = (1/T)e-tlr and for an Erlang distribution a = (4t/T
2
)e-
21
tr.
(b) Show that the probability that no arrivals occur in a timet chosen
at random is
U
0
(t) = (1/T) Jo"' dx J:'"' a(x + y) dy = (1/T) {" Ao(Y) dy
Compute U
0
(t) for the exponential and the Erlang arrival distributions, and
compare them with the postarrival probabilities obtained in part (a). Why does
U
0
= A
0
in one case and U o r" A o in the other?
Chapter 12
12-I. A molecule in a gas collides from time to time with another
molecule. These collisions are at random in time, with an average interval r,
the mean free time. Show that, starting at timet = 0 (not an instant of collision)
the probability that the molecule has not yet had its next collision at time t is
e-t!r. What is the expected time to this next collision? Show also that the
probability that its previous collision (the last one it had before time t = 0) was
earlier than time -Tis e-rlr. What is the mean time of this previous collision?
Does this mean that the average time interval between collisions is 2r? Explain
the paradox.
12-2. In interstellar space, the preponderant material is atomic
hydrogen, the mean density being about 1 hydrogen atom per cc. What is the
probability of finding no atom in a given cc? Of finding 3 atoms? How many
H atoms cross into a given cc, through one of its l-cm
2
faces, per second, if the
temperature is 1 K? If T is 1000K?
12-3. A closed furnace F (Figure P-8) in an evacuated chamber
contains sodium vapor heated to 1000K. What is the mean speed v of the vapor
atoms? At t = 0 an aperture is opened in the wall of the furnace, allowing a
collimated stream of atoms to shoot out into the vacuum. The aperture is closed
again at t = r. At a distance L from the aperture, a plate is moving with velocity
u, perpendicular to the atom stream, so that the stream deposits its sodium atoms
along a line on the plate; the position of the stream that strikes at timet hits the
line at a point a distance X = ut from its beginning. Obtain a formula for the
density of deposition of sodium as function of X along the line, assuming that
_____
f
FIGURE P-8
Problems 403
r (L/v), and find the value of X for which this density is maximum. Sketch
a curve of the density versus X.
12-4. The momentum distribution in a gas, which has particle density
(N /V), is
f(p) = (27rmkT)-
3
1
2
e-p'f
2
mkT(l + E cos a)
where E 1 and where a is the angle between p and the x axis.
(a) Compute the mean drift velocity U of the gas.
(b) How many atoms per second are crossing, in the positive x direc-
tion, a unit area of the y-z plane? How many are crossing in the negative
direction? How are these quantities related to Ux?
t
eWo
'---_, t .
..-- : -- exterzor
I
"'---metal surface
interior
FIGURE P-9
12-5. Most conduction electrons in a metal are kept from leaving the
metal by a sudden rise in electric potential energy, at the surface of the metal, of
an amount eW0, where W
0
is the electric potential difference between the inside
and the outside of the metal. Show that if the conduction electrons inside the
metal are assumed to have a Maxwell distribution of velocity, there will be a
thermionic emission current of electrons from the surface of a metal at tem-
perature T that is proportional to yT exp( -e W0/kT) (see Figure P-9). What
is the velocity distribution of these electrons just outside the surface? [The
measured thermionic current is proportional to T
2
exp( -ecp/kT), where cp < Wo;
see Problem 26-2.]
12-6. A gas of molecules with a Maxwell distribution of velocity at
temperature Tis in a container having a piston of area A, which is moving outward
with a velocity u (small compared to (v)), expanding the gas adiabatically (Figure
P-10). Show that, because of the motion of the piston, each molecule that strikes
u
t
v'
x_ I
0 I 0'

I
FIGURE P-10
404 Kinetic Theory
the piston with velocity v at an angle of incidence 8 rebounds with a loss of
kinetic energy of an amount 2mvu cos 8 (u v). Show that consequently the
gas loses energy, per second, by an amount -dU = PAu = P dV, where dV
is the increase in volume of the container per second.
12-7. Helium atoms have a collision cross section approximately equftl
to 2 X lQ-
16
cm
2
In helium gas at standard conditions (1 atm pressure, 0C),
assuming a Maxwell distribution, what is the mean speed of the atoms? What
is their mean distance apart? What is the mean free path? The mean free
time?
12-8. The Doppler formula for the observed frequency f from a source
moving with velocity v., along the line of sight to the observer is
f = fo[1 + (v.,/c)]
where fo is the frequency radiated when the source is at rest and c is the velocity
of light.
(a) What is the distribution in frequency of a particular spectrum line,
radiated from a gas at temperature T?
(b) What is the variance (Cf- f 0)
2
) of this radiated frequency? The
square root of the variance is called the breadth of the line.
(c) Atomic hydrogen and atomic oxygen are both present in a hot gas.
How much broader will the hydrogen lines be, compared to oxygen lines of
roughly the same frequency?
Chapter 13
13-l. A particle, moving vertically under the influence of gravity,
periodically strikes and rebounds elastically from a horizontal plane. Sketch
its phase-space trajectory between bounces. If particle 1 is released at t = 0 a
distance h above the plane, with no initial velocity, and if particle 2 is released
under similar conditions at time t:.t, how far apart in phase space are the two
particles when particle 2 is released? How far apart are they when particle 1
just reaches the horizontal plane? Assume that t:.t is small compared to the
time between bounces of each particle. If the two particles are released simul-
taneously, one at height h the other at height h + t:.h, how far apart in phase
space are they when the first particle hits the plane?
13-2. Use the Maxwell-Boltzmann distribution to show that, if the
atmosphere is at uniform temperature, the density p and pressure P a distance
z above the ground is exp( -mgz/kT) times p0 and Po, respectively (where g is the
acceleration of gravity). Express po and Po in terms of g, T, and M., the total
mass of gas above a unit ground area. Obtain this same expression from the
perfect gas law, P = pkT /m and the equation dP = - pg dz giving the fall-off
of pressure with height (assuming T is constant). Find the corresponding
expressions for p and Pin terms of z if the temperature varies with pressure in the
way an adiabatic expansion does, i.e., P = (p/C)-r, T = (Dp)-r-
1
, where
'Y = (Cp/C.) [see Eqs. (4-12)].
13-3. The collision cross section of an air molecule for an electron is
about lQ-
19
m
2
. At what pressure will 90 percent of the electrons emitted from
a cathode reach an anode 20 em away?
13-4. A gas-filled tube is whirled about one end with an angular
velocity w. Find the expression for the equilibrium density of the gas as a
function of the distance r from the end of the tube.
Problems
13-5(a) The phase-space distribution for the N atoms in a gas is
f(r, p) = (1 + "(X) (21T'mkT)-312e-p'l2mkT
405
where the origin of position coordinates is at the center of a rectangular box of
volume V, occupied by the gas. What is the pressure in the gas? Is the gas in
mechanical equilibrium if no forces are acting? What sort of force would have
to be applied for the gas to be in mechanical equilibrium? Assume that 'YX is
smaller than 1 everywhere inside the box.
(b) The phase-space distribution is changed to
f(r, p) = [21T'mkT(x)]-al2e-p
2
/2mkT(x)
T(x) = To(1 - "(X)
What is the pressure now? Is the gas in mechanical equilibrium?
(c) Finally, the distribution is changed to
f(r, p) = (1 + 'YX)[21T'mkT(x)]-a!2e-p'/2mkT(x)
where T_ = To(1 - "fX) as before. To the first order in the small quantity 'Y is
the gas m mechanical equilibrium? Is it in thermal equilibrium?
The atomic magnets in a paramagnetic crystal are not completely
free .to themselves along the applied magnetic field; there is a small energy
of ( 'Y per magnet, where 'Y/k is a fraction of a degree K) to the crystal
lattice. In the bound state the magnets cancel their polarization in pairs. In
the state the magnets can align themselves parallel or antiparallel to
the field (I:e., J = t. for each magnet). The behavior can be approximated by a
model havmg five different quantum states for each pair of magnets:
0-both magnets bound, energy e
0
= 0
1-unbound, both magnets parallel to field, e
1
= 2"( - 2
2-unbound, magnet 1 parallel, magnet 2 antiparallel, e
2
= 27
3-unbound, magnet 1 antiparallel, magnet 2 parallel, e
3
= 2'Y
4-unbound, both magnets antiparallel, e
4
= 2'Y +


Show that the magnetization of the crystal is
X = 2e--rlkr cosh
2kT
where mn is the Bohr magneton, N m is the total number of atomic mao-nets and
J.lo = Sketch or describe the dependence of 9J( on 4) when kT
when kT 'Y Over what range of T and 4) is Curie's law valid?
Chapter 14
14-l. A tube of length L = 2 m and of cross section A = lQ-4 m2
C02 at. nor:nal conditions of pressure and temperature (under these
conditiOns. the diffusiOn constant D for C0
2
is about w-s m2/sec). Half the
C02 contams radioactive carbon, initially at full concentration at the left-hand
zero concentration at the right-hand end, the concentration varying linearly
m between. What is the value of tc for C02 under these conditions? Initially,
406 Kinetic Theory
how many radioactive molecules per second cross the mid-point cross section
from left to right? [Use Eqs. (14-15).] How many cross from right to left?
Compute the difference and show that it checks with the net flow, calculated from
the diffusion equation (net flow) = -D(dn/dx). . .
14-2. A gas is confined between two parallel plates, one movmg
respect to the other, so that there is a shear the gas,. the .mean velocity
a distance y from the stationary plate bemg (3y m the x (Figure P-11).
Show that the zero-order velocity distribution in the gas IS
(1/27rmkT)
3
1
2
exp{ -(l/2mkT)[(Pz - m(3y)
2
+ p; + p;Jl
= fo(Pz - m(3y, Pu, p,)
Use Eq. (14-7) to compute f to the first order of approximation. Show that the
mean rate of transport of x momentum across a unit area perpendicular to theY
axis is
(N /V) Ctcf3/m) I I I dpz dpy dp, = (N /V)tcf3kT,
which equals the viscous drag of the gas per unit area of the which equals
the gas viscosity
77
times (3, the velocity gradient. Express 77 m terms T and
f. (mean free path) and show that the diffusion constant D of Eq. (14-16) IS equal
to (77/ p), where p is the density of the gas.

r -r-

x-
FIGURE P-11
14-3. A vessel containing air at standard conditions is with
x-rays so that about lQ-Io of its molecules are ionized. A uniform electnc. field
of 104 is applied. What is the initial net flux of Of IOns?
(See Problem 13-3 for the cross section for electrons; the cross. sectiOn .for the
ions is four times this. Why?) What is the ratio between dnft velocity and
mean thermal velocity for the electrons? For the ions? .
14-4. The free electrons in a partly ionized gas (plasma) have a density
(N,/V) and an equilibrium distribution
fo(p) = (1/V) (27rmkT)-312e-p'/2mkT.
An oscillating electric field (;t = {;t
0
eiwT is applied, where (;to is pointed in the x
direction, and is uniform throughout V.
Problems
407
(a) Show that the Boltzmann equation for the resulting distribution is
f + tc(ajjat) = fo- eeotceiwr(afo/apz)
and thus show that f must have the form f
0
(p) + g(p)eiwT.
(b) By equating coefficients of the time-dependent and the time-
independent terms, show that
g(p) = 11 (:;;) (1/v)fo(p)
where we have assumed that tc = /.(1/v).
(c) Compute the drift velocity Vz. Show that when wtc 1 it is in
phase with (;t and corresponds to Eq. (14-12). Show that when wtc 1 it is out
of phase with (;t and corresponds to the separate motion of each electron in an
oscillating field.
(d) Compute the joule heat production per unit volume. Explain its
dependence on frequency.
14-5. A paramagnetic gas is subjected to an oscillating magnetic field
l8 = l8o cos wt (l8 = J.toS)). The atomic magnets have quantized directions
being to the field, with moment imB and energy imBlB, or
ant1parallel, with moment -imB and energy +imBlB, where mB is the Bohr
magneton.
(a) Use the Boltzmann equation to find the first-order distribution
function f for orientation of the atomic magnets, corrected to allow for the time
variation of 18. Assume that imBlB kT, expand the zero-order Maxwell-
Boltzmann distributionfo in powers of (mBl8/2kT), and discard all powers higher
than the first.
(b) Calculate the magnetization ?In = J.toN M(moment) of the gas as a
function of time, to the first order in (mBlB/2kT).
(c) Compute the energy gained by the gas JS) d?m (and therefore lost
by the oscillator) per second, to the first order.
14-6. A plasma is subjected to an electric field (;tz in the x direction and
a magnetic field lB. in the z direction, both fields being independent of position
and time.
(a) Show that the Boltzmann equation for the distribution in momen-
tum of the electrons in the plasma is
f = fo - etc [{;tz (jf_) + ).8, Pv ( aj) _ ).8, Pz (!L)J
apx m apx m ap.
(b) Compute f to the second order in tc by setting f = f
0
+ tcf
1
+ t2/2
and equating coefficients of powers of tc up to the second. Show that if fo is the
Maxwell distribution, !1 = (e{;txPx/mkT) X fo and
!2 = [- (e
2
{;t;JmkT) + (e
2
{;t;p;/m
2
k
2
T
2
) - (e
2
{;txlBzPv/m2kT)]f
0
Why does the magnetic field have no first-order effect on the distribution?
(c) Compute the components Ux and Uv of the drift velocity. What is
the physical significance of the drift in the y direction?
. 14-7. It is suggested that the conduction electrons in a metal might be
considered to be a gas in thermal equilibrium with the atomic ions which are
held together in the crystal lattice. The electron collisions would be with
408 Kinetic Theory
the ions and the t, in the Boltzmann equation would .then be .the time between
collisions with the ions. In Chapter 26 we see that this model Is not a very good
approximation, but it is useful to see what it predicts. .
( ) Use the formulas of this chapter to compute the electric conduc-
tivity of silver (mol. wt. = 108, density 10,500 kg/m
3
, density ?f
ions = 6 x lQ2Bjm3) in terms of e, N,/V, t.., m, k, T, and (1/v). The electric
conductivity of silver at room temperatures is 7 X 1Q7 mhos/m. If we use the
Maxwell distribution to compute (1/v), what does this predict f?r the free
th f.. of the electrons in silver? How does this compare with the distance
b:tween the ions? How does the formula predict the conductivity should vary
with T? The actual conductivity is proportional to T-
1
at room temperatures
and to T-4 at lower temperatures (see page 361).
(b) Write out the formula for the heat conductivity of the
assuming they have a Maxwell distribution, in terms of same as I.n
(a). Show that the ratio between thermal and electric conductiVIty, for this
model, divided by T, is just (5k
2
/2e
2
), of T, t..
6
or. (1/v).
measured value of this quantity, for silver, for 250 K < T < 500 K, IS 2.3 10 .
What does the model predict? Below 150K the measured value of this ratio
decreases rapidly. .
(c) Use the formulas of this chapter to evaluate the thermoelectric
power, of Eqs. (8-47) et seq., for silver. .
14-8. Suppose the mean free path 'A of the electrons IS not
but is proportional to the electron's velocity v. How then would
depend on T? Does this suggest an answer to some of the discrepancies of
Problem 14-7?
Chapter 15
15-l. A solid cylinder of mass M is suspended from its center by a
fine elastic fiber so that its axis is vertical. A of the cylinder through
an angle (}from equilibrium requires a torque KO to the When. sus-
pended in a gas-filled container at temperature T, the cylmder rotatwnal
fluctuation due to Brownian motion. What is the deviati?n (C!.O) of
the amplitude of rotation and what is the standard dev1atwn (ll.w) of Its angular
velocity? What would these values be if the container we:e evacuated? .
15-2. An inductance L connected across the termmals of a capacitance
C has electromagnetic energy of oscillation
U = (Q
2
j2C) + (LP/2) I = (dQjdt)
Q being the charge on the capacitance plates. The circuit is in thermal equilib-
rium with its surroundings at temperature T. There is a thermal noise current
and a fluctuating charge in the capacitor.
(a) Calculate (Q
2
) and (P).
(b) Assuming C = 10-12 farad, what is the rms voltage across the
capacitor plates at T = 300K? . . . . .
15-3. A piston slides without frictwn m a cyhnder of mternal cross
section A, and encloses a perfect gas of N molecules. The force pushes the
piston downward has a magnitude F = P0A no matter what the displacement of
the piston. Also assume that F is large enough so that we can neglect the effect
of the mass of the piston.
Problems
40
(_a) W:hat is the equilibrium distance Xo of the piston from the bottor
of the cylmder, If the temperature of the gas is T?
(b) What additional force is required to displace the piston a distanc
x (small compared to Xo) from equilibrium? Express this force in terms of J:
Xo, N, and T. Calculate the potential energy of the displacement in terms of th
same quantities.
(c) The piston does not stay in its equilibrium position; it oscillate:
up down of the fluctuations of pressure of the gas of the cylinder
What IS the relative probability that the piston is displaced from equilibriurr
between x and x + dx?
. . . (d) What is the mean-square amplitude (x
2
) of its displacement frorr
eqmhbrmm? Express the result in terms of x
0
and N.
that if the Hamiltonian energy of a molecule depends on r
generalized. q or p in such a way that H ---7 'XI as p
01
q ---7 'XI, It Is possible to generalize the theorem on equipartition of energy tc
(
q iJH) = (p iJH) = kT
oq av op av
Verify that this reduces to ordinary equipartition when H has a quadratic
dependence on q or p. If H has the relativistic dependence on the momentum
H = c y' (p;, + + p;) + m2c2
show that

= =

= kT
1.5-5 . . Analyze the thermal oscillations of electromagnetic waves along
a conductmg Wire of length L. In this case of one-dimensional standing waves
the nth wave '':ill have the form cos(1rnxjL)eiwt, where w = 21rj = 1rnc/L, c
the wave velocity, and f the frequency of the nth standing wave. Show that the
number of allowed frequencies between f and f + df is (2Ljc) df and thus if
every wave has a mean energy kT, the energy content of the waves with
bet":een.J and f + df is (2LkT /c) df. If the wire is part of a trans-
m.ISSion lm.e, whiCh IS by its characteristic impedance, all the energy
Will be to thiS Impedance in a time (2L/c). Show, consequently, that
the power delivered to the terminal impedance, the thermal noise power in the
frequency band df at frequency j, is kT df if every wave is a derrree of and
if every degree of freedom carries a mean thermal energy kT. This formula
with experiment at frequencies less than about 1010 cycles per second
or so, but It cannot be correct for all frequencies, clear out to f ---7 'XI or else each
wire would contain an infinite amount of electromagnetic energy. (See Problem
25-5 for the answer.)
. 15-6. Obse:vations of the Brownian motion of a spherical particle of
radms 4.4 X 10-
7
m m water at 300K, which has viscosity 1J = w-a newton-
sec/m2 were made every 4 sec. The displacements in the x direction
0
= x(t)
x(t - 4) are tabulated for 25 readings, as follows: '
-5.8 +3.1 -1.0 -2.0 -1.5
+3.4 -0.2 +2.6 -1.9 +1.8
-1.8 -3.5 +0.3 -2.2 -0.2
-0.5 +1.3 +0.4 -0.4 +2.5
+0.5 +0.3 +0.6 +1.5 +1.9
410 Statistical Mechanics
Compute the mean value of o and its variance. How close is this distribution to
the normal distribution of Eq. (11-23)? Use Eq. (15-11) to compute Avogadro's
number N 0 from the data, assuming that R = kN o is known.
Chapter 18
18-l. A gas of N point particles, with negligible (but not zero) collision
interactions enclosed in a container of volume V, has a total energy U. Show
that the point for the gas may be anywhere on a surface in phase space
which encloses a volume [VaNi(27rmU)aNIj(3N /2) !]. For an ensemble of these
systems to represent an equilibrium state, how must the system points of the
ensemble be distributed over this surface?
18-2. A harmonic oscillator has a Hamiltonian energy H related to its
momentum p and displacement q by the equation
p' + (Mwq)' = 2MH
When H = U a constant energy, sketch the path of the system point in two-
dimensional phase space. What volume of phase space does it enclose? In the
case of N similar harmonic oscillators, which have the total energy U given by
N N
l p;' + l (Mwq;)' = 2MU
i=l i=l
with additional coupling terms, too small to be included, but large enough to
ensure equipartition of energy, what is the nature of the path traversed by the
system point? Show that the volume of phase space "enclosed" by this path is
(1/N!) (27rU /w)N.
18-3. Use the final result of Problem 18-2 to show that the entropy of
N distinguishable harmonic oscillators, according to the microcanonical ensemble
is
S = Nk[l + ln(kT /liw)]
18-4. A linear array of N particles is spaced equally along a straight
line. Suppose each particle has two possible states, which may be designated as
states A and B and that we have adjusted the energy origin so the A state has
energy - {3 and 'the B state has energy + {3. Suppose also that there is interaction
between nearest neighbors in the linear array, so that every pair with like states
(AA or BB) contributes 0 to the energy and every pair with unlike states (A!3 or
BA) contributes a . . (This is a. one-dirr:ensional model ma_gnets m a
ferromagnetic or antiferromagnetiC material, where state A IS onentatwn
to the field and B is anti parallel orientation, with {3 = ml8; the energy a Is the
difference between the coupling energy between neighboring antiparallel magnets
and that between neighboring parallel magnets. It could also be a model for
the mixing of two substances in a binary alloy; state A is the presence of one sort
of atom state B the presence of the other sort.)
'(a) Let m be the number of particles in state A (so that N - m
particles are in state B). These A particles can be placed anywhere along the
line they may be all at the beginning of the line, and all the B states may be at the
end' or there may be a group of A's at the beginning and at the end, with all the
B's 'grouped in the middle, and so on. Suppose the A's are grouped in n groups
Problems
411
(n .:::; m), each group comprised of one or more A's, separated from the nextAgrou
by a group of one or more B's. Show that the number of B groups is eith!
n - 1 or n or n + 1 that the number of unlike pairs (divisions between A
groups and B groups) IS either 2n - 2 or 2n- 1 or 2n, respectively. Show that
when N, n, and mare large enough, the energy of the linear array is U =
- 2{3m + 2om, where m 2: n.
(b) Use the results of Problem 11-5e to show that the number of dif-
ferent wa:ys in which the m states A may be arranged among the n different A
groups, at least in each group, is [m!/n!(m - n) !] and the
number of d.1fferent ways m whiCh the (N - m) B states are arranged within the
n B groups IS
[(N - m) !jn!(N - m - n) !].
. (c) Show entropy of a microcanonical ensemble having a
lmear array of m particles m state A and (N - m) particles in state Band having
the A states in n separated groups is
S = k[mlnm + (N- m) ln(N- m)- 2nlnn
- (m- n) ln(m- n) - (N- m- n) ln(N- m- n)]
(d) Show that application of Eq. (18-1) leads to the pair of equations
e2/3/kT = (N - m)(m- n) e2a/kT = (N- m- n)(m- n)
(N- m- n)m n'
which must be solved simultaneously to find equilibrium values of nand m and
thus of U and S, for a specified T.
(e) If f3 = 0 (zero applied magnetic field, if the system is a linear array
of magnets) show that the equilibrium value of m is itN, and those of nand S are
n = -----,-( N--'-/---'2)_
ea/kT + 1 S = Nk ln [2 - aN tanh(__!!__)
2kT 2T 2kT
What .is the most. likely of the states along the line when kT a?
Wh.at IS most likely ordermg for kT a, when a is positive (binding between
unhke pairs stronger than binding between like pairs)? When a is negative?
Chapter 19
19-l. A system consists of three distinguishable molecules at rest each
of which has a quantized magnetic moment, which can have its z
7-M, 0, or -ft!. Show that there are 27 different possible states of the system;
hst them all, givmg the total z component M,; of the magnetic moment for each.
Compute the entropy S = -k"'2J; X ln(f;) of the system for the following a
priori probabilities f;:
(a) All 27 states are equally likely (no knowledge of the state of the
system).
(b) Each state is equally likely for which the z component M of the
total magnetic moment is zero; J; = 0 for other states (we know that M, = 0).
(c) Each state is equally likely for which M, = M; J; = 0 for all other
states (we know that M, = M).
412 Statistical Mechanics
(d) Each state is equally likely for which M, = 3M; f; = 0 for all
other states (we know that M. = 3M).
(e) The distribution for which Sis maximum, subject to the require-
ments that 'Zj; = 1 and the mean component 'Zf;.M,; is equal to -yM. Show that
for this distribution
f; = exp[(3M - M,;)al/(1 + x + x
2
)3
where x = e"'M (a being the Lagrange multiplier) and where the value of x
(thus of a) is determined by the equation 'Y = 3(1 - x
2
)/(1 + x + x
2
). Com-
pute x and S for 'Y = 3, 'Y = 1, and 'Y = 0. Compare with a, b, c, and d.
19-2. Suppose we wish to set up an ensemble for a system depending
on the thermal variables T, Sand the mechanical variables Y, X [see Eqs. (8-17)
et seq.], and the system is quantized so we can write down the allowed values
JJ:, of the system's energy and the allowed values X, of the mechanical variable
X, for each quantum state v of the system. We prepare the ensemble as follows:
all systems in the ensemble have N particles, all have been brought to thermal
equilibrium at temperature T, and they all have been brought to mechanical
equilibrium at a specified value of the intensive variable Y.
(a) Show that the distribution function f must satisfy the following
requirements: S = -k'Zf, lnf, is maximum, subject to
'Zfv = 1 and
where X is the mean value of X,, averaged over the ensemble, satisfying the
equation of state for the specified values of Y and T.
(b) Show that the solution of these requirements is
Gv = U- TS- YX = -kTlnZv
S = -(aGv/aT)y X = -(aGvfaY)T
where Y and T are the independent variables.
19-3. A system contains N particles, confined in a box of volume Vat
temperature T. Each particle has one quantum state with zero energy, has
another one with energy k8, two more with energy k8 + kTo(Vo/V), three more
with energy k8 + 2kTo(V0/V), and so on, there being (n + 1) states having
energy k8 + nkTo(Vo/V). The lowest state is a bound state, with binding
energy k8; the unbound states have energies which spread apart as the volume
decreases.
(a) Use the canonical ensemble to show that the Helmholtz function
and the pressure of the system are
F = -NkTln {1 + (
1
e-DtT
1
)
2
}
- e ToVo TV
p = 2NkTo(Vo/V2)
(eToVoiTV - 1)(1 + eBIT(l - e ToVoiTV)2]
(b) Plot (PVo/NkT0) as a function of (V /Vo) for T = To and for
e
8
1To = 2, over the range 0 V 3V0 What are the limiting, approximate
formulas for F and P for V 0, for V ao ?
Problems
413
(c) What is the meaning of the sudden drop in p as V
tfhan i Vo? Is the system stable in this region? Does this imply a less
rom a bound phase to an unbound one? If so how p ase? ange
pressure at which this phase change occurs, the
(d) Plot the Gibbs function
(GINkTo) = (F /NkT
0
) + (PV INk To)
asafunctionof(V/Vo)forT = T
0
ande&/7'o = 2 C b' h' .

plot. of (PVo/NkTo) to a


o kTo), mstead of (VIVo). Explam the behavior of the curve.
Chapter 20
20-l. Show that a reasonable approximation to the value f
Eq. expressed as a function of the velocities c and f to wm, of
and long1tudmal waves, respectively, is t c1 o ransverse
Wm = (181!'W

+ (2lc;)]-I/3
The elastic constants for aluminum are: density 2700 kg/m3 b lk d
1
{3 = (1/K) - 7 5 X 1010 t I , u mo u us
l
. . - new ons m
2
, shear modulus a = 3 3 X 1010 Th t
ve oCitJes are related to the moduli by the t' 2 e wo
2 [ I ] equa wns c = [({3 + 4 ) 1 ] d


re speC! c eats.
20-2. For the solid described by Eq. (20-16) show that p _
(
[(VoV- V)/KVo] + (-yU n/V), wh<:Jre U = [(Vo _ V)212KV] + U d -
- 18) X (d8/dV) Th h o n an 'Y =
of temperature, the
f3 = (1IV)(aVIaT)p = K(aP/aT)v = (K-yC./V)
Constant 'Y is called the Griineisen constant.
20-3. When a crystal is stretched l f
of lattice vibrations in the direction of the the f:equency
Wo[
1
d +l

- Lo)
2
], where Lois the equilibrium length

:=
mo e , wit energy em
2N N
E, = (QAI2Lo)(L - Lo)2 + .2 hwon; + 2 hw.n;
=I i=I
of
the crystal volume). Calculate the equation of
(aF;aL)T, for temperatures larger than (hw /k) If g d '
temperature rise or fall when .crystal is oes the
.
1
20-4. Set up the partitwn function of Eq. (20-19) for the one d'
array of magnets discussed in Problem 18-4. Let E - 2 - Idmeln-
:JJlv = (2m - N){3 where {3 _ 1 "' v - an an et
I - 2J.1.0 mBq,.o
414
Statistical Mechanics
(a)
Show that the partition function is
(N- m)!m!
(n!)2(m- n)!(N- m- n)!
[
(2m- N)(:J- 2an] =
X exp kT
m,n
2: ef(m,n)
m,n
Write out the approximate for the exponent j(m, n), using Stirling's
approximation for the factorial functwns. . . _
(b) Expand the function j in a senes by settmg m -
j;N + x and n = {-N + y and using the Taylor serres
j(m, n) = f + xfm
+ yfn + j;x
2
fmm
(
N N) + 1 2, (N N) + . . .
+ xyfmn 2' 4 2Y Jnn 2' 4
where the subscripts indicate partial derivatives. Then from sum;at1on
over m, n to integration with respect to dm and dn, obtammg a secon -or er
expression, useful for kT >>a and (:J,
Zm c:::::2N exp[-(Na/2kT) + j;N(a/2kT)2 + j;N((:JjkT)2]
Compute the entropy and the magnetization to this approximation.
Chapter 21
21-l. Each of the N particles (distinguishable particles), in a. box of
volume v has two possible energies, 0 orE, where E inversely
the volume (E = b/V, where b = constant). What IS the equatwn of state o
this system? bl b
21-2. A system consists of a box of volume V and a varm. e num er
of indistinguishable (MB) particles each of mass m: Each particle can be
"created" by the expenditure of energy'}'; once created It becomes a
erfect gas of point particles within the volume V. The allo,:ed o e
stem are therefore n'}' plus the kinetic energies of n particles ms1de V, _for
;[ = 0, 1, 2, . . . . Show that the Helmholtz function for this system (canomcal
ensemble) is
F = kT In [I cvn.Xnjn!)] = -kTVX
n=O
where X = (21rmk/T jh2)312e-rtkT. Calculate the that n particles
are present in the box and thence obtain an expressiOn for N, the mean numbeJ
of free particles present as a function of /', T, and V. _Also calculate S, Cv, an
p from F and express these quantities as functions of N, T, and V.
Problems 415
21-3. A gas consists of a mixture of j;N molecules of mass M
1
and j;N
molecules of mass ll12, in a volume Vat temperature T. Set up the partition
function, assuming that the molecular rotation and vibration can be neglected
and that the molecules of each kind are indistinguishable from each other (but
distinguishable from those of the other kind). Calculate the entropy, internal
energy, and pressure of the mixture. Compare the formula for S with that of
Eq. (6-15).
Chapter 22
22-l. What fraction of the molecules of H2 gas are in the first excited
rotational state (l = 1) at 20K, at 100K, and at 5000K? What are the
corresponding fractions for 02 gas? What fraction of the molecules of H
2
gas
are in the first excited vibrational states (n = 1) at 20K and 5000K? What
are the corresponding fractions for 02 gas?
22-2. Plot the heat capacity Cv, in units of Nk for 0 2 gas, from 100 to
5000K.
22-3. The solid of Eqs. (20-14) sublimes at low pressure, at a sub-
limation temperature T, which is large compared to fJ, the resulting vapor being
a perfect diatomic gas, with properties given by Eqs. (21-14) and (22-2) (where
Orot << T, Ovib). Show that the equation relating T, and the sublimation
pressure P, is approximately
G. = VoP. + jNkfJ + 3NkT.ln(fJjT.) - NkT.
= Gu c::::: NkT,In(P. V0T/1
2
jN0kfJ
5
'
2
) - NkT.
where the equation
No= V
0
(47r1ekfJ/h
2
)(27rmkfJjh2)312
defines the constant N 0 Since Vo Vu = NkT./P. and fJ T., show that this
reduces to
P. c:::::Nok -YT0!Vo
Also show that the latent heat of sublimation is
L, = T,(Su - S,) c::::: j;NkT.
Chapter 23
23-l. At very high temperatures a gas is composed entirely of atoms of
mass m. In a container of volume V its grand partition function (MB statistics)
is
3 =
where x = (27rmkT/h
2
), and J.La is the chemical potential of the atom. At lower
temperatures some atoms combine in pairs to form diatomic molecules, each of
mass 2m, chemical potential J.Lm, and binding energy 'Y Assume that these
molecules have no rotational or vibrational states, so each molecule has energy-'}'
plus its translational, kinetic energy.
(a) Write out the expression for the grand partition function (MB
416
Statistical Mechanics
statistics) for the mixture of atoms and molecules, as a function of V, x, "(, J.l.a,
J.l.m, and kT. From this, obtain the grand p_otential Q = -PV and, by dif-
ferentiation, obtain the expressions for Na and N m, the mean number of atoms and
molecules in the system. _ _
(b) Write out the expression for the Gibbs function G = JJ.aNa + JJ.mN m
in terms of kT, Na, N m, "(, x, and V. For thermodynamic equilibrium at constant
P and T, what must be the ratio between J.l.m and J.l.a] _
(c) From this ratio compute the value of N m :,:sa of Na, V, x,
and ('Y/kT). Over what range_ of is f!m Na? What is the
equation of state (in terms of N = Na +_2Nm) m this range oJ
What is the equation of state in terms of N in the range where Nm << Na?
23-2. Work out the grand canonical ensemble for a gas of point atoms,
each with spin magnetic moment, which can have magnetic energy +iJJ.4'> or
_lJ.1.4) in a magnetic field 4'> in addition to its kinetic energy. Obtain the
for Nand expressions for Q, J.l., S, U, Cvroc, and the equation of state,
in terms of N, T, and 4). How much heat is given off by the gas when the
magnetic field is reduced from 4'> to zero isothermally, at constant volume?
Chapter 24
24-l. A system consists of three particles, each of which has three
possible quantum states, with energy 0, 2E, or 5E, r:spectively. _Write out. the
complete expression for the partition function Z for this system: (a) If the
are distinguishable (b) if the particles obey Maxwell-Boltzmann statistics;
(c) if they obey Ein'stein-Bose statistics; (d) if they obey Fermi-Dirac statistics.
Calculate the entropy of the system in each of the cases.
24-2. The particles of Problem 19-3 were distinguishable particles.
Work out the grand canonical ensemble for particles having the same energy
levels but different statistical behavior.
(a) Suppose the are MB particles. grand
potential, the mean number N, and the IJelmholtz Q + JJ.N. Show
that when F is expressed as a function of N, T, and V, It differs from the F of
Problem 19-3 by the term NkT(ln N - 1). Why?
(b) Suppose the particles are bosons. Show that
{ I x- 0- n(ToVo/V)]}
Q = kT In[1 - exiT] + kT
4
(n + 1) X In 1 - exp T
n=O
_ { [n(T0V0/V) + 0- x] }-
1
N = [e-xiT - 1]-1 +
4
(n + 1) X exp T - 1
n=O
where x = (JJ./k). Show that, when T < and N 1, the quantity x
-T[N- e-BIT(l - e-voTo)-2]-I, that Nb = N- N
1
of the particles are in the
bound state and N
1
e-BIT(l - e-voToiVT)-2 are in the upper, unbound states,
and that P ,.:,_, (N
1
kT /V) and G = kxN. Is there a phase transition in this range
ofT?
(c) Suppose the particles are fermions. Write out the expressions for
Problems
417
Q and N as functions of T, V, and x = JJ./k. By using the equations
no
l (n + 1)
n=O
and
no
l n(n + 1)
n=O
show that, when T < 0 and N 1, x (T0V
0
jV)(2N)112 and p INxk = IG
compare with the P of b. Is there a phase transition in this range J T?
3
'
Chapter 25
25-l. The maximum intensity perounit wavelength interval, in the sun's
spectrum, occurs at a wavelength of 5000 A. What is the surface temperature
of the sun, assuming the sun is a black body?
25-2. Show that, for Einstein-Bose particles (bosons)
S = -k l [n; ln(n;) - (1 + n;) In(1 + n,)]
i
2573: lot has been reported that the fission bomb produces a tempera-
ture of a K. Assuming this to be true over a sphere 10 em in diameter:
(a) IS the radiant-energy density inside the sphere? (b) What is the rate of
radiatiOn the surface? (c) is the radiant flux density 1 km away?
(d) What IS the wavelength of maximum energy per unit frequency interval?
. 25-4. The distribution can be obtained by considering each
standmg wave in a rectangular enclosure (LxLuLz) as a degree of
With coordmate Qp proportional to the amplitude of the electric vector
momentum Pp proportional to the amplitude of the magnetic vector, and
with a field energy, corresponding to a term in the Hamiltonian equal to
27rc
2
Pz + (w;/87rc
2
)Q;, where c is the velocity of light and where the allowed
frequency of the vth standing wave is given by
w; = 11"
2
C
2
[(kP/Lx)2 + (mp/Lu)2 + (nJ,)2]
(because of polarization, there are two different waves for each trio k., mv, n.).
Use t?e of Eqs: (20-4) to (20-11) to prove that the average energy
contamed m those standmg waves with frequencies between w and w + dw is
dJF = (h/7r
2
C
3
)w
3
- 1). Compare this derivation with the one dealing
With photons, whiCh produced Eq. (25-3).
25-5. Work out the correct form for the distribution-in-frequency of
the thermal energy of electromagnetic waves along a wire, discussed in Problem
15-5. Prov_e that the energy content of the waves with frequencies between f
and f + df IS
[2Lhf df/c(ehl/kT - 1)]
ar:d the between f and f + df which is delivered to a matching ter-
mmalimpedance IS [hf df/(ehllkT- 1)]. Show that this agrees with the results
?f for low frequencies. Above what frequency (for T = 300K)
IS the result in error? What is the total energy delivered per second
over all frequencies? '
. 25-6. A container of volume V has N short-range attractive centers
(potential wells) fixed in position within the container. There are also bosons
418 Statistical Mechanics
within the container. Each particle can either be bound to an attractive center,
with an energy -'Y (one level per center), or can be a free boson, with energy
equal to its kinetic energy, E. Use t!:e analysis of this to show
equation relating the mean number N of bosons to their chemical potential J.L IS
_ N (-J.L) (21rmkT)a'
2
N = 1 + No/It2 kT No= gV _h_2_
Draw curves for -J.L/kT as a function of No/N for NjN = 1 and for 'Y/kT = 0.1
and 1.0 using Table 25-1. Draw the corresponding curves for PV /NkT.
' 25-7. Suppose the particles of Problem 25-6 are MB particles instead
of bosons. Calculate the partition function Z for a canonical ensemble and
compare it with the Z for Problem 21-2.
Chapter 26
26-l. Show that, for Fermi-Dirac particles (fermions),
S = - l [n; ln(n;) + (1 - n;) ln(1 - ri;)]
i
26-2. The conduction electrons of Problem 12-5 are, of course, fermions.
Show that, for FD statistics, the thermionic emission current from the metal
surface at temperature Tis proportional to T
2
exp( -ecpjkT), where cp = Wo -
J.L Wo - J.Lo is called the thermionic work function of the surface.
26-3. The container and N attractive centers of Problem 25-6 have
N fermions, instead of bosons, in the system. By using Eqs. (26-2) and (26-3)
show that the equation relating J.L and T and Vis
-- N N ( /kT)
N - e<-y-,JtkT + 1 + o'Y/ J.L
N0 as in Problem 25-6
Plot J.L/kT as function of N0/N for 'YfkT = 0.1 and 1.0, using Table 26-1. Draw
the corresponding curves for PV /NkT.
26-4. A simple model for a nucleus is that of a Fermi gas, held together
by a harmonic oscillator potential. The allowed energies of each nucleon
are en = (n + j)hw (n = 0, 1, 2, ... ). The nth quantum level has Yn =
2(n + 2)(n + 1) distinct states. Then, start with the expression for the grand
potential
!J = -kT l Yn ln[1 +
n
(a) Calculate the expression for the average number of particles N.
(b) Show that this expression is a sum of terms rin. Sketch the plot of
rin as a function of En, when kT << J.L
(c) The energy-level spacing hw for a given nucleus is 10 Mev at
T = 0. Its Fermi energy J.LF = kTF turns out to equal 50 Mev. How many
nucleons are there in the nucleus?
(d) For 0 < T TF, the internal energy is
U(T) NkTF[f + (7r
2
/4)(TjTp)2 + .. ]
Problems
41
1
The nucleus of part c absorbs 50 Mev from an incident proton. What is it:
resultant temperature (give kT in Mev)?
(e! . What energy must an incident proton give up to the nucleus tc
transform 1t mto an MB gas?
. 26.-5. The equilibrium distribution function for conduction electron:
m a metal Is not the Maxwell distribution, but
fo = + lj-I
where e = (p; + + p;)/2m is the kinetic energy of one of theN electrons anc
fo dp': dp = 7n:fo d{3 sin ada :v(2me) de is the probability that the electron'
1
positiOn ISm a umt volume and Its is in the element dpx dpu dp. at p
(a) Evaluate the upper hmit J.LF of the Fermi distribution at T = 0 ir
terms of h, m, and the number (N /V) of conduction electrons per unit volume
(b) A uniform electric field & is applied to the metal in the x direction
Solve the Boltzmann equation
f + tc(ajjat) tc[(p/m) gradrfo + F gradpfo]
to obtain the first-order approximation for f in terms of tc, j
0
, (aj
0
fapx) =
(Px!m)(afo/ae), and the various constants .
. (c) Write out the integral expression for the resultant current I in thE
metal; mtegrate over the angles a and {3 and obtain the current as an integral
over e only, with the integrand containing (aj
0
jae).
[f f Px d{3 sin ada = 0; f f p;, d{3 sin ada = (87r/3)me.]
(d) Show that when kT J.LF,
Jo"' F(e)(afofae) -(2/Nh
3
)F(J.LF)
where F is any smoothly varying function of e. Use this to evaluate the inte,.ral
fo.r the current and thus obtain I in terms of N, e, &, tc, and m. Compare this
With the MB formula (14-13).
Chapter 27
27-l.
0 to 1.
Calculate the heat capacity of D2 as a function of T j Brat from
27-2. The Schrodinger equation for a one-dimensional harmonic
oscillator is
h2 a2ifl
Hif; =- __ + lmw2x2.t. = E'
2m ax2 'f' 'f'
Its allowed energies and corresponding wave functions are
En= hw(n + i)
if!n(X) = Hn(X vmwjh) exp( -mwx2j2h)
where Jl_o(z) :== 1, Ht(z) = 2z, H2(z) = 4z
2
- 2, H
3
(z) = sza- 12z, etc.
Two IdentiCal, one-dimensional oscillators thus have a Schrodinger equation
H(x, y)'Jf = - !:!_ + 'Jf + lmw2(x2 + y2)'Jf = E'Jf
2m ax
2
ay2
420
Statistical Mechanics
where x is the displacement of the first particle from equilibrium and Y that of
the second. .
(a) Show that allowed of this for the energy
hw(n + -!), may be written either as lmear combmatwns of products
1/lm(x)l/1,.-m(Y) for different values of m between 0 and n, or else as lmear com-
binations of the products
1/lm (x;.y) 1/ln-m (X0)
(b) Express the solutions

and

form = 0, 1, and 2 as linear combinations of the solutions 1/lm(x)if;,.(y) form, n =
0, 1, 2.
(c) Which of these solutions are appropriate if the two particles are
bosons? Which if they are fermions? . .
(d) Suppose the potential energy has an interpartwle repuls1ve term
_lmK2(x - y)2 (where K2 < w
2
) in addition to the term tmw
2
(X
2
+ Y
2
).
th!t, in this case, the allowed energies for bosons differ from those for fermwns.
Which lie higher and why?
REFERENCES
The texts listed below have been found useful to the writer of this volume.
They represent alternative approaches to various subjects treated here, or more
complete discussions of the material.
W. P. Allis and M. A. Herlin, "Thermodynamics and Statistical Mechanics,"
McGraw-Hill, New York, 1952, presents some alternative approaches.
H. B. Callen, "Thermodynamics," Wiley, New York, 1960, is a "postulational"
development of the subject.
J. D. Fast, "Entropy," McGraw-Hill, New York, 1962, is a readable discussion
of the entropy concept in thermal physics.
E. Fermi, "Thermodynamics," Prentice-Hall, Englewood, N.J., 1937, is a short,
readable discussion of the basic concepts.
T. L. Hill, "Statistical Mechanics," McGraw-Hill, New York, 1956, treats the
Maxwell-Boltzmann statistics in considerable detail, including the modern
theories of liquids and solids.
Allen L. King, "Thermophysics," Freeman, San Francisco, 1962, gives a number
of examples, taken from modern physics.
Charles Kittel, "Elementary Statistical Physics," Wiley, New York, 1958,
contains short dissertations on a number of aspects of thermodynamics and
statistical mechanics.
L. D. Landau and E. M. Lifchitz, "Statistical Physics," Addison-Wesley, Reading,
Mass., 1958, includes a thorough discussion of the quantum aspects of
statistical mechanics.
F. Reif, "Fundamentals of Statistical and Thermal Physics," McGraw-Hill
Book Co., New York, 1965, uses statistical mechanics from the beginning.
F. W. Sears, "Introduction to Thermodynamics, the Kinetic Theory of Gases,
and Statistical Mechanics," Addison-Wesley, Reading, Mass., 1953, also
provides some other points of view.
J. C. Slater, "Introduction to Chemical Physics," McGraw-Hill, New York, 1939,
has a more complete treatment of the application of statistical mechanics to
physical chemistry.
421

Potrebbero piacerti anche